LSAT and Law School Admissions Forum

Get expert LSAT preparation and law school admissions advice from PowerScore Test Preparation.

User avatar
 Dave Killoran
PowerScore Staff
  • PowerScore Staff
  • Posts: 5852
  • Joined: Mar 25, 2011
|
#46090
Complete Question Explanation
(The complete setup for this game can be found here: lsat/viewtopic.php?t=8712)

The correct answer choice is (E)

This question can be solved by either referring to the templates or by using the rules.

If the rules are used, the final two rules indicate that G and L must see different films. Consequently, answer choice (E) is proven correct.

If the templates are used, nothing in either template suggests that answer choice (A), (B), or (C) is correct. In answer choice (D), I, L, and V could all see the Hitchcock film under the 2-4-1. The templates ultimately show that answer choice (E) must be true.

Get the most out of your LSAT Prep Plus subscription.

Analyze and track your performance with our Testing and Analytics Package.